Which coupon is a better deal? 15℅ off of $75 shoes or $15 off of $75 shoes? Please show your work

Answers

Answer 1

Answer:

15 dollars off of 75 is better

Step-by-step explanation:

75 multiplied by 1.5 is only 11 dollars


Related Questions

A number divided by negative three is greater than negative three Write the inequality and solve 50 points i need the correct answer asap !!!! Will mark brainiliest !!!

Answers

Answer:

No clue, but i would recommend using desmos,  mathpapa, quizlet,

Step-by-step explanation:

The 3rd term of an arithinetic progression is-9
and the 7th term is -29. Find the 10th term of the
progression.​

Answers

Answer:

-64

Step-by-step explanation:

The computation of the 10th term of the progression is shown below:

The 3rd term is -9

A_n = a + (n - 1)d

-9 = a + (3 - 1)d

-9 = a + 2d

-9 - 2d = a

Now the 7th term is -29

A_n = a + (n - 1)d

-29 = a + (7 - 1)d

-29 = a + 6d

Put the a value to the above equation

-29 = -9 - 2d + 6d

-29 + 9 = 4d

-20 = 4d

d = -5

Now

-9 - 2d = a

-9 - 2(-5) = a

-9 - 10 = a

a = -19

Now finally the 10th term of the progression is

= a + (n - 1)d

= -19 + (10- 1)-5

= -19 -45

= -64

Could someone please help me, I have to turn this in, in 30 minutes, only answer if you know how to do it

Answers

9514 1404 393

Answer:

  a. The value of A₀ changes from 10 to 20

  b. t is replaced by (t-5)

Step-by-step explanation:

a. The given formula comes with a description that says A₀ is the initial dose in mg. If the initial dose changes from 10 mg to 20 mg, then A₀ changes from 10 to 20. The function becomes ...

  A = 20·0.97^t

__

b. Shifting the graph of the function to the right by 5 minutes is accomplished by replacing t with (t-5). At the same time, the domain changes from t ≥ 0 to t ≥ 5. The function becomes ...

  A = A₀·0.97^(t-5) . . . . . . . . for t ≥ 5

Which equation has a slope: 3/2, y-intercept: –4

Answers

Answer:

The equation of the line that has a slope of 3/2 and a y-intercept of -4 is:

y=3/2x-4.

Step-by-step explanation:

You want to find the equation for a line that has a slope of 3/2 and a y-intercept of -4.  First of all, remember what the equation of a line is:

y = mx+b  Where:  m is the slope, and  b is the y-intercept

All you really have to do here is  replace m with 3/2, which is the slope you gave, and  replace b with -4, the y-intercept you gave,  in the equation y=mx+b.

What's XY? Base A is 20 and base B is 36

Answers

Answer:

Step-by-step explanation:

c

A parachutist's rate during a free fall reaches 90 miles per hour. What is this rate in feet per second? At this rate, how many feet will the parachutist fall during 5 seconds of free fall? In your computations, use the fact that 1 mile is equal to 5 feet. Do not round your answers

Answers

Answer:

The parachutist's rate of fall is 132 feet per second. In turn, in 5 seconds, the parachutist will drop 660 feet.

Step-by-step explanation:

Given that a parachutist's rate during a free fall reaches 90 miles per hour, to determine what is this rate in feet per second and, at this rate, how many feet will the parachutist fall during 5 seconds of free fall, knowing that 1 mile is equal to 5280 feet, the following calculations must be performed:

90 x 5280 = 475,200

475,200 / 60/60 = X

7,920 / 60 = X

132 = X

Thus, the parachutist's rate of fall is 132 feet per second.

132 x 5 = 660

In turn, in 5 seconds, the parachutist will drop 660 feet.

Determine whether the two expressions are equivalent.If so,tell that property is applied. (37) and (2.3). 7*
O Yes; Identity Property
O Yes; Associative Property
O Yes; Commutative Property
O Yes; Inverse Property
O Expressions are not equal

Answers

Answer:

Yes; Inverse Property...

I hope it helps..

The answer is inverse property i hope this helped u

How many miles will Mrs. Garcia drive in eight hours if she is traveling 65 mph? _ miles

Answers

Answer:

520 miles

Step-by-step explanation:

65 mph * 8 hours = 520 miles

I hope this helps :)

In a random sample of 70 undergraduate students from a large university, it was found that 37 had used library books. Find the sample proportion

Answers

Answer:

The sample proportion of undergraduate students that used library books is 0.5286.

Step-by-step explanation:

The sample proportion is the number of desired outcomes divided by the size of the sample.

In this question:

Sample of 70 undergraduate students, 37 had used library books.

[tex]p = \frac{37}{70} = 0.5286[/tex]

The sample proportion of undergraduate students that used library books is 0.5286.

An angle measures 60 degree it is divided into 5 angles with equal measures. What is the measure of those angles

Answers

Answer:

Each part of the angle would measure 12 degrees

Step-by-step explanation:

Brainlest and points need help math

Answers

Answer: 43.96in^2
Explanation:
14x3.14=43.96in^2

Help please !!! Thanks guys

Answers

Answer:
(b+4) and (b-1)
Explanation:
do a generic rectangle
b times b is b^2
b times 4 = 4b
-1 times b = -1b
-1 times 4 = -4
b^2+4b-1b-4
simplify
b^2+3b-4

Science (from the Latin word Scientia, meaning "knowledge") is a systematic enterprise that builds and organizes knowledge in the form of testable explanations and predictions about the universe.
The earliest roots of science can be traced to Ancient Egypt and Mesopotamia in around 3500 to 3000 BCE

Answers

Answer:

?

Step-by-step explanation:

What are you even asking-

In Exercises 45–48, let f(x) = (x - 2)2 + 1. Match the
function with its graph

Answers

Answer:

45) The function corresponds to graph A

46) The function corresponds to graph C

47) The function corresponds to graph B

48) The function corresponds to graph D

Step-by-step explanation:

We know that the function f(x) is:

[tex]f(x)=(x-2)^{2}+1[/tex]

45)

The function g(x) is given by:

[tex]g(x)=f(x-1)[/tex]

using f(x) we can find f(x-1)

[tex]g(x)=((x-1)-2)^{2}+1=(x-3)^{2}+1[/tex]

If we take the derivative and equal to zero we will find the minimum value of the parabolla (x,y) and then find the correct graph.

[tex]g(x)'=2(x-3)[/tex]

[tex]2(x-3)=0[/tex]

[tex]x=3[/tex]

Puting it on g(x) we will get y value.

[tex]y=g(3)=(3-3)^{2}+1[/tex]

[tex]y=g(3)=1[/tex]

Then, the minimum point of this function is (3,1) and it corresponds to (A)

46)

Let's use the same method here.

[tex]g(x)=f(x+2)[/tex]

[tex]g(x)=((x+2)-2)^{2}+1[/tex]

[tex]g(x)=(x)^{2}+1[/tex]

Let's find the first derivative and equal to zero to find x and y minimum value.

[tex]g'(x)=2x[/tex]

[tex]0=2x[/tex]

[tex]x=0[/tex]

Evaluatinf g(x) at this value of x we have:

[tex]g(0)=(x)^{2}+1[/tex]

[tex]g(0)=1[/tex]

Then, the minimum point of this function is (0,1) and it corresponds to (C)

47)

Let's use the same method here.

[tex]g(x)=f(x)+2[/tex]

[tex]g(x)=(x-2)^{2}+1+2[/tex]

[tex]g(x)=(x-2)^{2}+3[/tex]

Let's find the first derivative and equal to zero to find x and y minimum value.

[tex]g'(x)=2(x-2)[/tex]

[tex]0=2(x-2)[/tex]

[tex]x=2[/tex]

Evaluatinf g(x) at this value of x we have:

[tex]g(2)=(2-2)^{2}+3[/tex]

[tex]g(2)=3[/tex]

Then, the minimum point of this function is (2,3) and it corresponds to (B)

48)

Let's use the same method here.

[tex]g(x)=f(x)-3[/tex]

[tex]g(x)=(x-2)^{2}+1-3[/tex]

[tex]g(x)=(x-2)^{2}-2[/tex]

Let's find the first derivative and equal to zero to find x and y minimum value.

[tex]g'(x)=2(x-2)[/tex]

[tex]0=2(x-2)[/tex]

[tex]x=2[/tex]

Evaluatinf g(x) at this value of x we have:

[tex]g(2)=(2-2)^{2}-2[/tex]

[tex]g(2)=-2[/tex]

Then, the minimum point of this function is (2,-2) and it corresponds to (D)

I hope it helps you!

turn 3/40 into a fraction

Answers

3/40 is a fraction itself.
ANSWER:
3/40 is already a fraction...
if you meant decimal then it’s 0.075

sorry if this doesn’t help

The height of a cuboid whose volume is 385 cu cm and base area is 35 sq cm is​

Answers

Answer:

11cm

Step-by-step explanation:

Given data

Volume of cuboid =385 cm^3

Base area= 35cm^2

We know that

Volume = Area* Height

substitute and solve for Height

385=35*H

H= 385/35

H= 11cm

Hence, the height is 11cm

The owner of a local nightclub has recently surveyed a random sample of 300 customers of the club. She would now like to determine whether or not the mean age of her customers is over 35. If so, she plans to alter the entertainment to appeal to an older crowd. If not, no entertainment changes will be made. Suppose she found that the sample mean was 35.5 years and population standard deviation was 5 years. What is the p-value associated with the test statistic

Answers

Answer:

0.0416

Step-by-step explanation:

Given :

Sample size, n = 300

Sample mean, x = 35.5

Population mean, m = 35

Standard deviation, s = 5

The test statistic :

Zstatistic = (x - m) / s/sqrt(n)

Zstatistic = (35.5 - 35) / 5/sqrt(300)

Zstatistic = 0.5 / 0.2886751

Zstatistic = 1.732

Using the p value calculator from Zstatistic :

One tailed P value at 95% confidence interval is : 0.0416

Help pls and thank you ..

Answers

Answer:

i think this is the anwer 100

Ms. Gallegos burns 236 calories riding her bike each hour. She wants to burn more than 590 calories riding her bike at the same rate.



Which inequlity represents all possible values for t, the number of hours Ms. Gallegos must ride her bike to burn more than 590 calories?



A.
t > 2.5

B.
t < 2.5

C.
t > 0.4

D.
t < 0.4

Answers

Answer:

Bro the answer is A

Step-by-step explanation:

trust me straight 100% brain dead person

(30, points)
A right triangle is shown. Which is closest to the value of an x​

Answers

Answer:

I think the answer might be 13. I don't know it is wrong or not

THIS IS SO BAD RN SMH HELP ME WITH ANOTHER MATH SOLVING
4 x 3 x 4 x 9 + 3 - 20

Answers

Answer:

=415

Step-by-step explanation:

The girl in that picture is so pretty oml so if its you then you pretty a.ff

two packages of rice cost the same amount. Which is the better buy? 12 ounce or 400 grams

Answers

Answer:

400 grams

Step-by-step explanation:

12ounce= 340.19 grams approximately

So,its best to buy the package that weighs 400 grams

Without performing the calculations, which expression equals the greatest value?
A)
2x (3-5)
B)
3x (
3-5)
)
5x (3-5)
D)
7x (3-5)

Answers

A 2x(3-5)

due to pemdas whatever is inside the parentheses goes first. it’s going to be a negative. if you make the outside number greater, the value of the overall number will become less because negative numbers are less in value the higher they are

WILL GIVE BRAINLIEST. NEED URGENTLY!

Use the FOIL method to find the product below.
(x + 3)(x2 - 6x)
A. x3 - 9x2 - 18
B. X8 - 9x2 - 18x
c. x3 - 3x2 - 18x
D. x2 – 3x2 - 18

Answers

Answer:

C) x³-3x²-18x

Step-by-step explanation:

x³-6x²+3x²-18x

x³-3x²-18x

From a collection of 52 store customers, 3 are to be chosen to receive a special gift. How many groups of 3 customers are possible?

Answers

Answer:

132,600 possibilities

Step-by-step explanation:

The first one chosen can be 1 of 52

The 2nd one chosen is 1 out 51

And the 3rd chosen is 1 out 50

So...

52 x 51 x 50 = 132,600

There are 22,100 groups of 3 customers that are possible from the given collection of 52 store customers.

What is a combination?

It is an arrangement of a set of numbers from a total set where the order of the set is not relevant.

We have,

The number of groups of 3 customers that can be chosen from a collection of 52 store customers can be calculated using the combination formula:

nCr = n! / r!(n - r)!

where n is the total number of customers (52) and r is the number of customers chosen (3).

Substituting the values, we get:

52C3 = 52! / 3!(52 - 3)!

52C3 = 52! / 3!49!

52C3 = (52 × 51 × 50) / (3 × 2 × 1)

52C3 = 22,100

Therefore,

There are 22,100 groups of 3 customers that are possible from the given collection of 52 store customers.

Learn more about combination here:

https://brainly.com/question/14355408

#SPJ2

there are 60 sixth graders at wilson middle school. only 50% of the sixth graders will attend the morning assembly. how many sixth graders will be at the morning assembly?

Answers

Answer:

30 sixth graders

Step-by-step explanation:

50% = 0.50

multiply 60 by the decimal fraction of the percentage:

60 x 0.50 = 30

you can also just take 60 and divide by 2 since 50% means half

Color blindness is an inherited characteristic that is more common in males than females. Let F represent female, and C represent red-green color blindness.

P(C) = 0.039 P(F and C) = 0.004 P(F or C) = 0.548



2. Use the Addition Rule or the "OR" formula for probability, substitute the values given, and solve for the probability of a Female.

P(F or C) = P(F) + P(C) - P(F and C)

Answers

Answer:

0.513 = 51.3% probability of being a female.

Step-by-step explanation:

We are given the following formula:

[tex]P(F \cup C) = P(F) + P(C) - P(F \cap C)[/tex]

In which:

Event F: Female

Event C: Red-green color blindness.

Probability of a Female

We have to find P(F), which is given by:

[tex]P(F) = P(F \cup C) + P(F \cap C) - P(C)[/tex]

We are given by:

[tex]P(C) = 0.039, P(F \cap C) = 0.004, P(F \cup C) = 0.548[/tex]

So

[tex]P(F) = P(F \cup C) + P(F \cap C) - P(C) = 0.548 + 0.004 - 0.039 = 0.513[/tex]

0.513 = 51.3% probability of being a female.

22:4 :: x : 16, what is the value of x, if all the four numbers are in proportion.​

Answers

Answer:

= 22:4 :: x : 16

22*16 = 4x

352 = 4x

x = 352/4

x = 88

Hope this helps you. Do mark me as brainliest.

PLEASE HELP!!!!!! ILL GIVE BRAINLIEST *EXTRA 40 POINTS** !! DONT SKIP :((

Answers

the answer is no. the solution is (1.333, 7.333)

Answer: No, it is not a solution

Step-by-step explanation:

Substitute the value of x as 1 and y as 6

y = x + 6

6 = 1 + 6

6 = 7 (no)

-----------------------------------------

y = 4x + 2

6 = 4(1) + 2

6 = 4 + 2

6 = 6 (Yes)

-----------------------------------------

Even though the solution works in the second equation, it does not work in the first

Therefore (1,6) is not a solution to the system of equations

3.14 x 6.25 step by step

Answers

Answer:

Step-by-step explanation:

3.14x6.25

19.625

Answer:

19.625

Step-by-step explanation:

Other Questions
A number has three digits. The digit in the hundreds place is the difference of 4 and 2. The digit in tje tens place is the sum of 2 and 3. The digit in the ones place is more than the digit in the tens place. What is the number? Your manager has a live work early for the day so your coworker takes it upon herself to boss everyone around. What is the source of conflict?A. GoalsB. RolesC. Values D. Resources Which of the following pairs of groups of elements cannot form ionic bond? *IA & VIIAIA & VAVIIA & VIIIAIIA & VIA Provide an example of a universal theme.Please write your answer in a complete sentence.lol again please help Sharon made $45 in interest by placing $500 in a savings account with simple interest for 1 year. What was the interest rate? Isabella is travelling in Brazil during the summer, She knows that a 15-minute phone call home to her family costs $45.85 and a 20-minute phone call costs $58.30 sabella only has $55.81 to spend on her phone call. How long can she talk to her family? what are polygastric animals and how do their digestive system aid in digesting roughage What did Roosevelt, Churchill, and Stalin agree to at the Yalta Conference?They called for a conference to discuss forming a new international organization, the United Nations.They decided the best way to defeat Japan was to launch an invasion rather than drop an atomic bomb.They approved a plan for the Soviet Union to occupy Germany, Italy, and Japan after the war.They approved a negotiated peace with Germany that would allow Hitler to remain in office. What did Greece do to protect themselves after the wars were over? HELP WILL GIVE BRAINLIEST Given an expression can you write an equivalent form of the expression and explain the properties you used? area is the amount of .......a figure covers ? There are 320 students in a school.There are 36 more boys than girls.What fraction of the students in the school are boys? Which statement best defines economic policy?O it is a set of actions taken by interest groups on issues that relate to the economy.O It is a set of laws written by a government that relate to citizens' quality of life.O It is a set of laws written by officials about earning and spending money in a government. O It is a set of actions taken by a political party to raise economic issues during an election. What is the measure? She finally became even more violent in her opposition than her husband himself. She was not satisfied with simply doing as well as he had commanded; she seemed anxious to do better. In the above sentence, the word commanded comes from a Latin root word that means to A callB trustC joinD order A car travels a distance of 190 miles in a time of 3.4 hours.What is its average speed rounded to 1dp? Enseguidal Your teachers constantly tell you what to do. What dothey tell you?1-escribir / composicin2- poner / atencin3.-no comer / en clase4.-llegar / a tiempo5.- escuchar / bien6.-usar / boligrafo7.- sacar / buenas notas8.- no hablar / tanto9.- recordar / fecha10.- no salir / temprano PLS HELP!! I will make brainliest!!Which process is an example of a chemical reaction?a. an iron nail rustingb. bath water cooling while you take a bathc. a piece of metal being heated until it expandsd. a glass window breaking when hit with a baseball Which of the following terms relate to hair color or hair loss? Choose all that apply.melanocytephaeomelaninsudoriferous glandscomodoneseumelanintelogen effluviumcochineal An archer uses 2000J of energy to pull a bow string 0.15m. What is the force constant (k) of the bow string?